Đến nội dung

Chris yang nội dung

Có 231 mục bởi Chris yang (Tìm giới hạn từ 26-04-2020)



Sắp theo                Sắp xếp  

#704191 Giải hệ $\left\{\begin{matrix} x_5+x_2=m...

Đã gửi bởi Chris yang on 24-03-2018 - 08:41 trong Phương trình - hệ phương trình - bất phương trình

Giải và biện luận hệ phương trình sau:

$\left\{\begin{matrix} x_5+x_2=mx_1\\ x_1+x_3=mx_2\\ x_2+x_4=mx_3\\ x_3+x_5=mx_4\\ x_4+x_1=mx_5\end{matrix}\right.$




#691515 Tìm điều kiện của $m$ để đồ thị hàm số $y=x+\sqrt{mx...

Đã gửi bởi Chris yang on 25-08-2017 - 16:07 trong Hàm số - Đạo hàm

Tìm điều kiện của $m$ để đồ thị hàm số $y=x+\sqrt{mx^2+x+1}$ có tiệm cận đứng.




#665308 $3^x5^{x^2}=375$

Đã gửi bởi Chris yang on 20-12-2016 - 23:01 trong Phương trình - hệ phương trình - bất phương trình

Cho $x\in\mathbb{R}$ thỏa mãn $3^x5^{x^2}=375$. Nếu $-x=\log_ab$ thì $a+b$ có thể nhận giá trị nào sau đây?

A. $20$               B. $25$               C. $15$                           D. $18$




#665306 $\int\frac{dx}{2^{x}+1}$

Đã gửi bởi Chris yang on 20-12-2016 - 22:57 trong Tích phân - Nguyên hàm

Tìm A = $\int\frac{dx}{2^{x}+1}$

Đặt $t=2^x+1$ thì $dt=\ln 2 (t-1)dx$

$\Rightarrow A=\frac{1}{\ln 2}\int \frac{dt}{t(t-1)}=\frac{1}{\ln 2}\int \left ( \frac{1}{t-1}-\frac{1}{t} \right )dt=\frac{1}{\ln 2}(\ln|t-1|-\ln|t|)+c$




#658675 Cho $k$ là số nguyên dương. Chứng minh rằng tồn tại các số nguyên...

Đã gửi bởi Chris yang on 21-10-2016 - 16:39 trong Số học

Cho $k$ là số nguyên dương. Chứng minh rằng tồn tại các số nguyên $x, y$ không số nào chia hết cho 3 sao cho $x^{2}+3y^{2}=3^{k}.$

$x^2=3^k-3y^2$ chắc chắn chia hết cho $3$ rồi còn gì @@




#658153 Tìm tất cả các số nguyên dương $(q, r, p),$ với $p$ là số...

Đã gửi bởi Chris yang on 16-10-2016 - 22:02 trong Số học

Tìm tất cả các số nguyên dương $(q, r, p),$ với $p$ là số nguyên tố thỏa mãn $2^{q}+r^{2}=2p.$

Dễ thấy $p$ lẻ

Từ PT suy ra $r$ chẵn $\Rightarrow 4|r^2$. Thấy $2^{q-1}=p-\frac{r^2}{2}$ lẻ do $p$ lẻ nên $q-1=0\Rightarrow q=1$

Khi đó $r^2=2(p-1)\Rightarrow p-1=2^{2k+1}x^2$ với $k,x\in\mathbb{N}$ hay $p$ có dạng $2^{2k+1}x^2+1$ ( Theo nguyên lí Dirichlet về sự tồn tại vô số số nguyên tố dạng $ak+b$ thì $p$ có vô số giá trị thỏa mãn $3,19,73,163,883....$)

 Vậy $(p,q,r)=(2^{2k+1}x^2+1,1,2^{k+1}x)$ với $k,x\in\mathbb{N}$




#658073 Cho $S(n)$ là tổng các chữ số của $n$. Tìm $n$...

Đã gửi bởi Chris yang on 16-10-2016 - 16:49 trong Số học

Tính chất cơ bản: Ước nhỏ nhất (khác $1$) của một số $a$ là một số nguyên dương không vượt quá $\sqrt{a}$. 

Từ đây, suy ra nếu $S(n)$ là ước nguyên dương lớn nhất khác $n$ thì $S^2(n)\geq n$ Suy ra nếu $n$ có $t$ chữ số, khi đó điều kiện cần là $(9t)^2\geq 10^{t-1}$. Khi đó dễ CM $t<5$ bằng quy nạp.

+) Nếu $t=4$, $n=\overline{a_1a_2a_3a_4}$, $n\leq 36^2=1296$ nên $a_1=1\Rightarrow n\leq (1+9+9+9)^2=784$ ( vô lý)

+) Nếu $t=1$ thì hiển nhiên vô lý.

+) Nếu $t=2$ Đặt $n=\overline{a_1a_2}$. Cần có $10a_1+a_2=k(a_1+a_2)$, trong đó $k$ là số nguyên tố nhỏ hơn $\sqrt{99}$, tức là $k\leq 9$

Thay $k=2,3,5,7$ vào $\rightarrow a_1,a_2...$

+) Nếu $t=3$ có $\overline{a_1a_2a_3}\leq 729$ nên $a_1\leq 7\rightarrow n\leq (6+9+9)^2=576\Rightarrow a_1\leq 5\rightarrow n\leq 22^2=484$ nên $a_1\leq 4$

Giờ chỉ cần thử các giá trị $a_1=1,2,3,4$ và tiếp tục làm như TH $t=2$ nhưng với biến $a_2,a_3$ ta sẽ tìm được $n$ thỏa mãn.




#658013 Tìm các số a,b sao cho $a^b=b^a$

Đã gửi bởi Chris yang on 16-10-2016 - 00:08 trong Số học

Tìm các số số tự nhiên $a,b$ sao cho $a^b=b^a$

Với $a=b\in\mathbb{N}$ thì bài toán luôn đúng. Xét $a\neq b$. Giả sử $a>b$.

Đặt $a=p_1^{m_1}p_2^{m_2}....p_k^{m_k}$ với $p_i\in\mathbb{P}$ và $m_i\in\mathbb{N}$. Từ đó kéo theo $b=p_1^{n_1}p_2^{n_2}....p_k^{n_k}$

Vì $a^b=b^a$ nên $\frac{a}{b}=\frac{m_i}{n_i}=\prod_{1}^{k}p_i^{m_i-n_i}>1$

Giả sử $p$ là ước nguyên tố lớn nhất của $a$ và $b$, hiển nhiên $\frac{m}{n}\geq p^{m-n}\Rightarrow m\geq np^{m-n}$

$\Rightarrow m-n\geq n(p^{m-n}-1)\geq p^{m-n}-1$ $(\star)$

Ta sẽ CM với $x\geq 1\in\mathbb{N}$, $p\in\mathbb{N}^*$ thì $p^x\geq (p-1)x+1$ $(1)$ bằng quy nạp. Giả sử điều này đúng với $x=t$, tức là $p^t\geq (p-1)t+1$, khi đó $p^{t+1}=p.p^t\geq p(p-1)t+p=(p-1)(t+1)+1+t(p-1)^2\geq (p-1)(t+1)+1$, tức là điều này đúng với cả $x=t+1$, do đó $(1)$ được CM. Dấu $=$ xảy ra khi $x=1$

 

Quay trở lại bài toán, với $m-n\geq 1$, ta có $p^{m-n}\geq (p-1)(m-n)+1$. Kết hợp với $(\star)$ suy ra $p=2$ thỏa mãn kéo theo. Dấu $=$ xảy ra khi $m-n=1$. Hơn nữa ta cũng thu được $\frac{m}{n}=2$ nên $m=2,n=1$, hay $(a,b)=(4,2)$

Vậy $a=b$ hoặc $(a,b)=(4,2)$ và các hoán vị




#657909 GPT: $\sqrt[3]{x+6}+\sqrt{x-1}=x^{2...

Đã gửi bởi Chris yang on 15-10-2016 - 14:30 trong Phương trình - hệ phương trình - bất phương trình

2 câu bôi đỏ nhé.

Câu 9:

ĐKXĐ: $1\leq x\leq 3$. 

Ta có bài toán tương đương $2\sqrt{x-1}+2\sqrt{x-3}\leq 2x^3-8\sqrt{2x^3}+20\Leftrightarrow 2(\sqrt{x-1}+\sqrt{3-x})\leq (\sqrt{2x^3}-4)^2+4$

Thấy rằng $(\sqrt{2x^3}-4)^2+4\geq 4$ và theo BĐT Cauchy-Schwarz thì $2(\sqrt{x-1}+\sqrt{3-x})\leq 2\sqrt{(x-1+3-x)(1+1)}=4$. DO đó BPT luôn đúng với mọi $x$ nằm trong khoảng xác định, tức là $1\leq x\leq 3$

Câu 1: (chưa hoàn thành)

 

Đặt $x-1=a,y+1=b$.( $a,b>0$).Ta có $a^b=b^a$

$\Rightarrow b\log a=a\log b\Rightarrow \frac{\log a}{a}=\frac{\log b}{b}$. Đặt $b=a^r$ với $r\in\mathbb{R}$ thì $\frac{\log a}{a}=\frac{\log a^r}{a^r}=\frac{r\log a }{a^r}\Rightarrow r=a^{r-1}\Rightarrow a=r^{\frac{1}{r-1}}\Rightarrow b=r^{\frac{r}{r-1}}$

Đặt $\frac{1}{r-1}=t$, xét TH ta sẽ thu được bộ $(a,b)=(x-1,y+1)=\left ( 1+\frac{1}{t} \right )^{t},\left (1+\frac{1}{t} \right )^{t+1}$

Từ PT $(1)$ ta sẽ thu được $\sqrt{y+1}\geq 2\rightarrow y\geq 3$. Việc cần làm là ta đi chứng minh hàm $\left (1+\frac{1}{t} \right )^{t+1}$ là hàm nghịch biến trên hai khoảng, sẽ thu được $t\leq 1$. Lại có hàm $\left (1+\frac{1}{t} \right )^{t}$ là hàm đồng biến trên hai khoảng, kéo theo $x-1\leq 2\rightarrow  x\leq 3$ Mà $3,5\geq x\geq 3$ nên $x=3$, kéo theo $y=3$.

 

P.S: Cái này là ý tưởng của mình, chưa hoàn thành vì mình không biết nó đúng hay sai. Bạn nào thạo về phần đạo hàm, phương trình xem hộ mình xem ý tưởng của mình đúng hay không hộ cái

 

 

 




#651055 $\sqrt{x}+\sqrt[3]{x+7}=\sqrt[4]...

Đã gửi bởi Chris yang on 24-08-2016 - 13:10 trong Phương trình - hệ phương trình - bất phương trình

Giải phương trình $\sqrt{x}+\sqrt[3]{x+7}=\sqrt[4]{x+80}$




#649948 Nhận và khoe quà của Diễn đàn 2016

Đã gửi bởi Chris yang on 16-08-2016 - 21:18 trong Góc giao lưu

Em nhận được quà rồi ạ :D

 

Nói chung là VMEO IV đối với em là một trải nghiệm tuyệt vời! Dù đã bỏ toán ngót 1 năm , và trong cuộc thi này vẫn chưa dành tâm huyết hết mình cũng như sắp xếp được thời gian nhưng kết quả lần này cũng coi như không uổng phí 3 năm học toán hsg của em ^_^ Chưa bao giờ em nghĩ sẽ có một kỉ niệm nào đó gắn bó với VMF cho đến khi quyết định tham gia cuộc thi. Thực sự cảm ơn BTC, món quà rất tuyệt vời! :)

Hình gửi kèm

  • DDTH1.jpg



#649642 Tính $F_{\text{đh}}$

Đã gửi bởi Chris yang on 14-08-2016 - 18:26 trong Các môn tự nhiên (Vật lý, Hóa học, Sinh học, Công nghệ)

Một con lắc lò xo treo thẳng đứng, vật treo ở dưới, chiều dài tự nhiên $l_0=50$ cm, Đưa vật lên vị trí lò xo không biến dạng rồi buông nhẹ, vật dao động điều hòa với với chu kỳ $0,5$ s. Lực cực đại mà lò xo tác dụng lên điểm treo là $5$ N. Khi chiều dài của lò xo là $54$ cm thì lực đàn hồi tác dụng lên vật có độ lớn là bao nhiêu?




#647652 $(2y-1)\sqrt{1+x}+(2y+1)\sqrt{1-x}=2y$

Đã gửi bởi Chris yang on 02-08-2016 - 17:07 trong Phương trình - hệ phương trình - bất phương trình

Giải HPT trên tập số thực $\left\{\begin{matrix} \sqrt{9y^2+(2y+3)(y-x)}+4\sqrt{xy}=7x\\ (2y-1)\sqrt{1+x}+(2y+1)\sqrt{1-x}=2y\end{matrix}\right.$




#646843 Tìm các số nguyên tố p,q

Đã gửi bởi Chris yang on 28-07-2016 - 00:56 trong Số học

tại sao 

 

dòng thứ 4 từ dưới lên là sao ạ

em không hiểu ạ

Cái đó em đọc phần "Số chính phương mod p" để hiểu rõ hơn.




#645760 Trung bình cộng của các ước của số $p^{m}q^{n...

Đã gửi bởi Chris yang on 21-07-2016 - 00:53 trong Số học

Cho $p,q$ là 2 số nguyên tố phân biệt. Chứng minh rằng: tồn tại các số nguyên dương $m, n$ sao cho trung bình cộng của các ước của số $p^{m}q^{n}$ là $1$ số nguyên.

ĐKĐB tương đương với tồn tại $m,n$ để $A=\frac{(p^{m+1}-1)(q^{n+1}-1)}{(p-1)(q-1)(m+1)(n+1)}=\frac{(p^m+p^{m-1}+...+1)(q^n+q^{n-1}+...+1)}{(m+1)(n+1)}\in \mathbb{Z}$

+) Nếu $p=2$ và $q$ lẻ:  chọn $n=q$ thì $n+1|q^n+q^{n-1}+...+1$, ta chọn $m+1=\frac{q^{n}+q^{n-1}+...+1}{n+1}=1+q+q^2+...+q^{n-1}$. Khi đó $A\in\mathbb{Z}$. TH $q=2$ và $p$ lẻ tương tự như vậy.

+) Nếu cả $p,q$ đều lẻ, chọn $m=p,n=q$ hiển nhiên ta thu luôn được $A\in\mathbb{Z}$ :D

Do đó luôn tồn tại $m,n\in\mathbb{Z}^+$ thỏa mãn điều kiện trên.




#645751 Tìm n nguyên dương thoả mãn $3^{n-1}+5^{n-1}|3^n+5^n...

Đã gửi bởi Chris yang on 20-07-2016 - 22:38 trong Số học

Bài 1:

 Tìm $n\in\mathbb{N^*}$ lẻ sao cho với $\left\{\begin{matrix} a,b|n \\ (a,b)=1 \end{matrix}\right.$ thì $a+b-1|n$

Bài 2:

 Tìm a,b,c nguyên dương thoả mãn $(a-1)(b-1)(c-1)|abc-1$

Bài 3:

 Tìm n nguyên dương thoả mãn $3^{n-1}+5^{n-1}|3^n+5^n$

 

Bài 1: Đặt $n=p^ta$ với $(p,a)=1$ và $p\in\mathbb{P}$, theo đkđb ta có ngay $p^t+a-1|p^ta\Rightarrow p^t+a-1|p^{2t}-p^t\rightarrow p^{2t}\geq 2p^t+a-1\Leftrightarrow p^{2t}\geq 2p^t+\frac{n}{p^t}-1>2\sqrt{2n}-1\Rightarrow p^{2t}\geq 2\sqrt{2n}$. Vì $p$ bất kỳ nên suy ra nếu $n$ có nhiều hơn hoặc bằng $4$ ước nguyên tố thì vô lý. Do đó ta xét các TH:

 

TH1: $n=p^t$ thì $a,b$ có một số là $1$. Giả sử đó là $b$ thì điều kiện trên luôn đúng

 

TH2: $n=p^t.q^m$ thì $p+q-1|p^t.q^m\Rightarrow p+q-1=p^{u}q^{v}$ với $u\leq t, v\leq m$ .Nếu $u,v\geq 1$ thì ta thu được $p=q=1$ (vô lý). Do đó một trong hai số $u,v$ bằng $0$. Giả sử $p>q$ Ta thấy $u=0$ vì nếu $v=0$ bắt buộc VT<VP. Do đó $p+q-1=q^v$. Lại có $m\geq v\geq 2$ nên $p+q^2-1=p^xq^y$. Bằng cách lập luận tương tự ở trên ta CM được rằng $x<2, x\neq 0\rightarrow x=1$  suy ra $q-1=q^{y-1}(p-q^{v-y})$, suy ra $y=1\Rightarrow p=q+1\Rightarrow p=3,q=2$, ta tìm được $n=12$

 

TH3: $n=p^tq^mr^s$. Giả sử $p>q>r$. Ta có $pq+r-1=p^{t'}q^{m'}r^{s'}$ ( $t'\leq t, m'\leq m, s'\leq s$ ) Lập luận tương tự TH2, thử lần lượt $a,b=0$,  ta thu được $pq+r-1=r^c$ $(1)$

Lại có $c\geq 2$ nên $pq+r^2-1=p^aq^br^c$ ( $a\leq t,b\leq m, c\leq s$), tương tự TH2 ta cũng có $pq+r^2-1=pqr^c$ $(2)$

Với $(1)$ và $(2)$ ta không tìm được bộ $(p,q,r)$ thỏa mãn, tức là không tồn tại $n$

 

Vậy $n=p^t$ hoặc $n=12$

 

Bài 2

Ta có $1\leq \frac{abc-1}{(a-1)(b-1)(c-1)}<\frac{abc}{(a-1)(b-1)(c-1)}\leq 2.2.2=6$

Không mất tính tổng quát giả sử $a\leq b\leq c$. Trước tiên dễ thấy TH $abc-1=(a-1)(b-1)(c-1)$ vô lý. 

Với các TH $abc-1=k(a-1)(b-1)(c-1)$ trong đó $k=2,3,4,5$ : ta giả sử $a\leq b\leq c$. Nếu $a>5$ kéo theo $b,c>5$, khi đó $\frac{abc-1}{(a-1)(b-1)(c-1)}\leq 1$ nên  $a=2,3,4$. Đến đây chỉ cần chịu khó xét các TH và thay giá trị của $a$ vào là có thể giải quyết dễ dàng.

 

Bài 3: 

ĐK tương đương $3^{n-1}+5^{n-1}|2.5^{n-1}\Rightarrow 3^{n-1}+5^{n-1}=2.5^t$ ( do $3^{n-1}+5^{n-1}$ chẵn) ( $t\leq n-1$)

Nếu $t\leq n-2$ thì hiển nhiên $\text{VT}>\text{VP}$ nên $t=n-1$, kéo theo $3^{n-1}=5^{n-1}$, suy ra $n=1$




#645465 Tìm $n$ để tổng các chữ số của $3n^2+n+1$ bằng 2016

Đã gửi bởi Chris yang on 18-07-2016 - 22:56 trong Số học

Với mỗi số tự nhiên $n$, đặt $f(n)$ là tổng  các chữ số của $3n^2+n+1$

 

a. Tìm giá trị nhỏ nhất của $f(n)$

 

b. Tìm $n$ để $f(n)=2016$

Giải như sau:

a) 

Nếu $f(n)=1$ thì $3n^2+n+1=10^k$ với $k\geq 1$. Điều này hiển nhiên vô lý vì $3n^2+n+1$ luôn lẻ

Nếu $f(n)=2$ , ta xét 2 TH sau:

 +) TH1: $3n^2+n+1=2.10^k$: tương tự phía trên <vô lý>

 +) TH2:  $3n^2+n+1=10^a+10^b$ với $a>b\in\mathbb{N}$. Do $3n^2+n+1$ lẻ  nên $b=0$, hay $3n^2+n=n(3n+1)=10^a=2^a.5^a$. Phương trình tích với $(n,3n+1)=1$ ta dễ dàng thu được phương trình vô nghiệm

Nếu $f(n)=3$. Ta xét TH sau: $3n^2+n+1=2.10^k+1\Rightarrow n(3n+1)=2.10^k$. Dễ dàng giải PT trên ta thu được $n=8$ thỏa mãn, nghĩa là phương trình trên có nghiệm, hay tồn tại $n$ sao cho $f(n)=3$ là min

Vậy $f_{min}(n)=3$

b)  

Đoạn này cũng không biết cách làm nào khác ngoài mò :D

Dễ thấy $f(n)=2016=3n^2+n+1\pmod 9$ nên suy ra $n\equiv 5\pmod 9$

Khi đó thử đặt $n=10^t-5$. Ta có $3n^2+n+1=3.10^{2t}-29.10^t+71=2999...9710000...0071$ bao gồm một chữ số $2$, $t-2$ chữ số $9$, $2$ cặp $71$ và $t-2$ chữ số $0$

$\Rightarrow f(n)=2+9(t-2)+2(7+1)=2016$ kéo theo $t=224$

Vậy $n=10^{224}-5$ thì $f(n)=2016$




#644228 Chứng minh tồn tại số $k\in N^{*}$

Đã gửi bởi Chris yang on 09-07-2016 - 15:08 trong Số học

CMR: Tồn tại số $k\in N^{*}$ sao cho 3k có chữ số tận cùng là 001 

Bài này sử dụng định lý Dirichlet

Xét  dãy gồm $1001$ số $3^{a_1},3^{a_2}....,3^{a_{1001}}$ khi chia cho $1000$ có khả năng dư $0,1,2,....,999$ ( gồm $1000$ số dư) nên theo nguyên lý Dirichlet tồn tại ít nhất $\left \lfloor \frac{1001}{1000} \right \rfloor+1=2$ số thuộc dãy trên có cùng số dư. Giả sử đó là $3^{a_i}$ và $3^{a_j}$ ( $a_i<a_j$) thì $3^{a_i}(3^{a_j-a_i}-1)$ chia hết cho $1000$, hay $3^{a_j-a_i}-1\equiv 0\pmod {1000}$, tức là tồn tại số $k=a_j-a_i$ sao cho $3^k$ có tận cùng là $001$  




#643925 Tìm các số nguyên tố p,q

Đã gửi bởi Chris yang on 07-07-2016 - 01:12 trong Số học

Tìm các số nguyên tố p.q sao cho: p^2-pq-q^3= 27

Phương trình $p(p-q)=(q+3)(q^2-3q+9)$

Dễ thấy $(p,q)=(7,2)$ là nghiệm. Nếu cả $p,q$ đều lẻ

TH1: $p|q+3\Rightarrow q+3\geq p\Rightarrow q^2-3q+9>q+3>p-q$, kéo theo $\text{VT}\leq \text{VP}$ ( vô lý)

TH2: $p|q^2-3q+9$. Đặt $q^2-3q+9=pk\rightarrow p-q=k(q+3)$. Ta thu được $q^2-q(k^2+k+3)+(9-3k^2)=0$. Với $k=1,2$ thì $q=3\Rightarrow p=9\not\in\mathbb{P}$.

Xét $k>2$, ta thấy $k|(2q-3)^2+27$, do đó nếu $r$ là ước nguyên tố lẻ nào đó của $k$ thì $\left ( \frac{-27}{r} \right )=\left ( \frac{-3}{r} \right )=1\Rightarrow r\equiv 1\pmod 6$, kéo theo $k\geq 7$. Nếu $k=7,8,9$ thì đều vô lý. Xét $k\geq 10$ 

Phương trình có nghiệm khi mà $T=(k^2+k+3)^2+4(3k^2-9)=k^4+2k^3+19k^2+6k-27$ là scp, với $k\geq 10$ dễ thấy $(k^2+k+8)^2<T<(k^2+k+9)^2$, do đó $T$ không thể là scp

Vậy $(p,q)=(7,2)$ 




#641279 $x^{2}+y^{2}+1=3xy$

Đã gửi bởi Chris yang on 19-06-2016 - 17:21 trong Số học

chứng minh tất cả nghiệm nguyên dương của phương trình $x^{2}+y^{2}+1=3xy$  là $(x,y)=(F_{2k-1},F_{2k+1})$

với  $F_{n}$ là dãy Fibonacci

Giải như sau:

 

Với dãy Fibonacci ta có hai tính chất quen thuộc là $F_{m+n}=F_{m-1}F_n+F_mF_{n+1}$ và $F_{n+1}=F_{n}+F_{n-1}$

Từ đó, biến đổi ta suy ra $F_{2k+1}=3F_{2k-1}-F_{2k-3}$.Xét dãy $(a_n)$ xác định bởi $ \left\{\begin{matrix}a_1=1,a_2=1\\ a_{n+1}=3a_n-a_{n-1}\end{matrix}\right.$

 

Ta sẽ chứng minh tất cả nghiệm $(x,y)$ của phương trình $x^2+y^2+1=3xy$ sẽ thuộc dãy trên. Dễ thấy $x=y=a_1=a_2=1$ là nghiệm nhỏ nhất. Theo định lý Viete, nếu $(x,y)$ là nghiệm của phương trình trên thì $(3y-x,y)$ cũng là nghiệm. Tương đương với khi ta có $(a_1,a_2)$ là nghiệm thì $(3a_2-a_1,a_2)$ hay $(a_2,a_3)$ cũng là nghiệm..... Cứ tiếp tục như vậy ta có $(a_n,a_{n+1})$ là nghiệm của phương trình trên.  

 

Giờ ta chỉ cần chỉ ra không có nghiệm nào của phương trình nằm ngoài dãy $(a_n)$

 

Phản chứng: Giả sử tồn tại $(x,y)$ cũng là nghiệm cả $(\star)$ nhưng thuộc dãy trên. Những cặp như thế này thuộc tập tạm gọi là $S$. Theo nguyên tắc cực hạn ta chọn được một cặp $(x_i,y_i)$ sao cho $x_i+y_i$ nhỏ nhất. TH $x=y$ chỉ cho ra 1 cặp $x=y=1$ thuộc dãy $(a_n)$ nên giả sử $x>y$. Xét PT $x^2-3xy_i+y_i^2+1=0$. Áp dụng định lý Viete ta có:

$\left\{\begin{matrix}x_i+x_i'=3y_i\\ x_ix_i'=y_i^2+1\end{matrix}\right.\Rightarrow x_i'=3y_i-x_i=\frac{y_i^2+1}{x_i}<x_i$ ( do $x_i>y_i$)

Suy ra bộ $(3y_i-x_i,y_i)\not\in S$ vì tính nhỏ nhất của $x_i+y_i$, tức là $(3y_i-x_i,y_i) $ thuộc dãy $(a_n)$, hay tồn tại $k$ sao cho $3y_i-x_i=a_k,y_i=a_{k+1}$

$\Rightarrow x_i=3a_{k+1}-a_k=a_{k+2}$ ( vô lý vì bộ $(x_i,y_i)\not\in (a_n)$

 

Do đó ta có đpcm




#640603 $\sum \frac{a}{\sqrt{4a^2+ab+4b^2...

Đã gửi bởi Chris yang on 15-06-2016 - 22:54 trong Bất đẳng thức và cực trị

PP tiếp tuyến hầu hết chỉ cho ta đc vế bậc 1 thôi a, còn nếu ra đc vế có cả phân thức thế kia thì thực sự là tiếp tuyến ko làm nổi 

Thực sự là nếu dùng PP tiếp tuyến cũng khó mò mẫm ra BĐT đẹp thế kia =))

Cái này chắc do kĩ năng sử dụng BĐT phụ trong giải BĐT. Ta luôn có BĐT Vasc quen thuộc là $\sum \frac{x+1}{x^2+x+1}\leq 1$ $(*)$với $xyz=1$ . Cho nên khi biến đổi BĐT về dạng như thế kia thì khả năng nghĩ đến BĐT $(*)$ là có khả thi :)

Như thế thì sau này với các bài BĐT dạng này có thể giải hướng kiểu kia được không nhỉ? :D Bài giải của bạn trên tuyệt quá!




#640396 Cho $n$ là một số nguyên dương lẻ. Chứng minh rằng $((n-1)^n+1...

Đã gửi bởi Chris yang on 14-06-2016 - 23:53 trong Số học

Cho $n$ là một số nguyên dương lẻ. Chứng minh rằng $((n-1)^n+1)^2\mid n(n-1)^{(n-1)^n+1}+n$.

Bài này rất đơn giản: Vì $n$ lẻ nên $(n-1)^n+1$ lẻ. Gọi $p$ là ước nguyên tố bất kỳ của $(n-1)^n+1$. Ta cần có

$2v_p[(n-1)^n+1]\leq v_p(n)+v_p[(n-1)^{(n-1)^n+1}+1]$

$\Leftrightarrow 2v_p[(n-1)^n+1]\leq v_p(n)+v_p[(n-1)^n+1]+v_p[\frac{(n-1)^n+1}{n}]=2v_p[(n-1)^n+1]$

Điều này luôn đúng nên ta có đpcm




#640177 $\sum \frac{a^3+1}{b^2+c^2} \geq a+b+...

Đã gửi bởi Chris yang on 14-06-2016 - 00:28 trong Bất đẳng thức và cực trị

Mình chứng minh tương đương thì ra nhưng rất dài còn về phần cm theo bđt phụ chưa nghĩ ra mọi người đưa ra ý kiến đi !

 

:ukliam2:  :ukliam2:  :ukliam2:  :ukliam2:  :ukliam2:  :ukliam2:  :ukliam2:  :ukliam2:  :ukliam2:  :ukliam2:

Thỉnh cách CM tương đương của bạn! Mình sử dụng cũng có quá trình biến đổi tương đương nhưng không dài cho lắm =))

 

Ta đưa về BĐT đồng bậc, tức là với $a,b,c>0$, cần chứng minh

 

$\frac{a^3+abc}{b^2+c^2}+\frac{b^3+abc}{c^2+a^2}+\frac{c^3+abc}{a^2+b^2}\geq a+b+c$

 

$\Leftrightarrow \left ( \frac{a^3+abc}{b^2+c^2}-a \right )+\left ( \frac{b^3+abc}{c^2+a^2}-b \right )+\left ( \frac{c^3+abc}{a^2+b^2}-c \right )\geq 0$

 

$\Leftrightarrow \frac{a(a-b)(a-c)}{b^2+c^2}+\frac{b(b-c)(b-a)}{c^2+a^2}+\frac{c(c-a)(c-b)}{a^2+b^2}\geq 0$ $(\star)$

 

Giờ thì giả sử $a\geq b\geq c$ thì $\frac{a}{b^2+c^2}\geq \frac{b}{c^2+a^2}$

 

Khi đó, BĐT $(\star)$ hiển nhiên đúng theo BĐT Vornicu- Schur

 

Dấu $=$ xảy ra khi $a=b=c=1$




#640065 Đề thi tuyển sinh vào lớp 10 THPT chuyên Vĩnh Phúc năm học 2016-2017 (vòng 2)

Đã gửi bởi Chris yang on 13-06-2016 - 16:01 trong Tài liệu - Đề thi

Bài 2:

a) Phương trình tương đương $(\sqrt{4x-3}-2x)^2=x^2\Leftrightarrow (\sqrt{4x-3}-x)(\sqrt{4x-3}-3x)=0$ 

Dễ dàng giải PT trên ta thu được $x=1,3$ là nghiệm

b)(Tính chia hết áp dụng cho cả số âm và dương)

Dễ thấy $y|x$ nên đặt $x=ty$. Ta có $t^2=ty+y^4+2y^2$. Gọi $m=\gcd(t,y)$ nên $t=t_1m,y=y_1m$ với $(t_1,y_1)=1$ $(1)$

Khi đó $t_1^2=y_1(t_1+y_1^3m^2+2y_1)$, kết hợp $(1)$ suy ra $y_1=\pm 1$

TH1 $y_1=1$ thì $t_1^2=t_1+m^2+2\rightarrow (2t_1-2m-1)(2t_1+2m-1)=9$

TH2: $y_1=-1$ thì $(2t_1-2m+1)(2t_1+2m+1)=9$

Hai phương trình tích trên đều có thể giải nghiệm nguyên dễ dàng!

 

Bài 4:

a) 

Do tứ giác $BMDF$ và $MECD$ nội tiếp nên $\angle BFD=\angle BMA=\angle DMC=\angle DEC$ $(1)$

Và tứ giác $ABCD$ nội tiếp nên $\angle FBD=\angle DCE$ $(2)$

Từ $(1), (2)\Rightarrow \bigtriangleup BDF\sim \bigtriangleup CDE$ 

$\Rightarrow \angle FDB=\angle EDC$. Mà $\angle FDB=\angle FMB$ và $\angle EDC=\angle EMC$ nên $\angle FMB=\angle EMC$, kéo theo $\overline{F,M,E}$

b) Từ $A$ kẻ tiếp tuyến $Ax$ của $(O)$ ( $x$ nằm cùng phía $B$ so với $OA$)

Ta có $\angle xAF=\angle ACB=\angle BDA=\angle AFM=\angle AFE$ ( do $\overline{F,M,E}$ và $BMDF$ nội tiếp)

$\Rightarrow Ax\parallel EF$. Hiển nhiên $Ax\perp OA\rightarrow OA\perp EF$

c) 

Để $PQ\parallel BC$ cần có $\frac{BQ}{QN}=\frac{PC}{PN}$

Theo tính chất đường phân giác $\frac{BQ}{QN}=\frac{BF}{FN},\frac{PC}{PN}=\frac{EC}{EN}$, nên cần chứng minh $ \frac{BF}{FN}=\frac{EC}{EN}\Leftrightarrow \frac{BF}{CE}=\frac{FN}{EN}=\frac{AF}{AE}=\frac{AC}{AB}$

$\Leftrightarrow BF.AB=AE.AC\Leftrightarrow AB.AF+AC.AE=AB^2+AC^2$

Thấy rằng $AB.AF=AE.AC=AM.AD=AM^2+AM.MD=AM^2+MB.MC=AM^2+\frac{BC^2}{4}$ nên ta đi chứng minh $AB^2+AC^2=2AM^2+\frac{BC^2}{2}$

Đến đây kẻ đường cao $AH$ của tam giác $ABC$ rồi sử dụng định lý Pitago ta có đpcm

 

 

 

 

 




#639578 Tìm nghiệm tự nhiên của phương trình: $(x-1)!+1=x^k$

Đã gửi bởi Chris yang on 11-06-2016 - 13:49 trong Số học

Chỗ $(****)$ mình không hiểu ? sao lại $x \leq x-1$

Mình viết nhầm, $k<x$ nên $k\leq x-1$